10 votos

¿Existe una cuarta componente del campo eléctrico y del campo magnético?

La pregunta

Si los tres campos eléctricos y magnéticos vectoriales provienen de la cuatrapotencial de cuatro componentes, ¿existe una cuarta componente del campo eléctrico y magnético?

Pregunta relacionada

He publicado la siguiente pregunta: ¿Cuál es el significado físico de la transformación dipolar de las ecuaciones de Maxwell? . Recuerdo que hace tiempo escribí las ecuaciones de Maxwell y crucé los campos con el vector de posición y pude transformar las ecuaciones de Maxwell de una fuente de carga monopolar a algo que parecía ser una ecuación de campo de origen dipolar. Cuando hice esto reveló un extraño cuarto componente en las ecuaciones de campo que creo que podría estar relacionado con esta pregunta,

14voto

Chris Kobrzak Puntos 46

En realidad, los campos eléctricos y magnéticos de una combinado tensor llamado tensor de campo electromagnético . Este es un tensor de rango 2 y tiene la forma* $$ F^{\mu\nu}=\left(\begin{array}{cccc} 0 & -E_x & -E_y & -E_z \\ E_x & 0 & -B_z & B_y \\ E_y & B_z & 0 & -B_x \\ E_z & -B_y & B_x & 0 \end{array}\right) $$ Tiene las siguientes propiedades:

  1. Es antisimétrico (por lo que $F^{12}=-F^{21}$ )
  2. Es sin rastro
  3. Tiene 16 elementos, pero sólo 6 valores distintos
  4. Cuando se multiplica por su tensor dual ( $G^{\mu\nu}$ ) da un valor invariante de Lorentz de $4\mathbf{B}\cdot\mathbf{E}$
  5. El producto interior, $F_{\mu\nu}F^{\mu\nu}=2(B^2-E^2)$ también es un invariante de Lorentz

También se pueden derivar las ecuaciones de Maxwell a través del tensor aplicando $\partial_\mu$ a ella. La ley de Gauss y la ley de Ampere provienen de $$ \partial_\mu F^{\mu\nu}=4\pi J^\nu $$ donde $J^\mu=\left(\rho,\,\mathbf{j}\right)$ es la cuatro-corriente. La ley magnética de Gauss y la ley de Faraday provienen de aplicar la La identidad de Bianchi para conseguir $$ \partial_\gamma F_{\mu\nu} + \partial_\mu F_{\nu\gamma} + \partial_\nu F_{\gamma\mu}=0 $$ O más concisamente, $$\partial_{[\mu}F_{\nu\gamma]}=0$$


Soy astrofísico, así que uso unidades cgs; en el SI, todos los campos eléctricos tienen un factor de $1/c$ .

6voto

Sean Bannister Puntos 141

Para responder a esta pregunta, es necesario tener una comprensión geométrica completa de las ecuaciones de Maxwell y de lo que representan.

Las ecuaciones de Maxwell son un sistema de EDP muy común. En notación STA, es simplemente

$$\nabla F = -J$$

Damos por sentado que $F$ es un bivector, y por tanto tiene 6 componentes, y que $J$ es un vector, y por tanto tiene 4 componentes. Pero esta ecuación describe hasta ocho ecuaciones separadas. ¿Por qué?

Para un campo bivectorial arbitrario $K$ la derivada $\nabla K$ puede tener términos vectoriales y trivectoriales. Que las ecuaciones de Maxwell sólo tienen un vector El término fuente es en realidad bastante significativo: forma parte del contenido físico de las ecuaciones de Maxwell. Estamos diciendo que el campo EM está determinado sólo por una corriente vectorial.

¿Qué pasaría si hubiera un término de fuente de corriente trivectorial? Sería una carga "magnética" (monopolos magnéticos) y una corriente asociada. Así que enseguida podemos apreciar lo que denotaría ese término fuente.

Pero espera, ¡hay más! Supongamos que tenemos corrientes eléctricas y magnéticas. ¿Qué tipos de campos podrían producirlas?

Como se ha tratado de conseguir, estos son las otras dos componentes de un campo que podrían entrar en esta ecuación diferencial. Son un campo escalar y un campo pseudoescalar. No estoy familiarizado con cómo se manifestarían estos campos, o qué harían.


Entonces, ¿por qué no lo descubrimos?

Dejemos que $\lambda$ sea el campo escalar. ¿Cómo afectaría esto a las ecuaciones de Maxwell con sólo un término de fuente de corriente?

Dejemos que $F = e_0 E + B$ donde he denotado implícitamente que el campo magnético es un bivector . Se puede identificar como un vector en su lugar y luego considerar $\epsilon_3 B$ pero el efecto neto es bastante mínimo.

Las ecuaciones de Maxwell se descomponen entonces como

$$\nabla \cdot F = \partial_t E - e_0 \nabla_3 \cdot E + \nabla_3 \cdot B = -\rho e_0 - j$$

y

$$\nabla \wedge F = -e_0 \nabla_3 \wedge E - e_0 \partial_t B + \nabla_3 \wedge B = 0$$

Añadir un campo escalar $\lambda$ afectaría sólo a la parte vectorial, con su gradiente:

$$-J = \nabla \cdot F - e_0 \partial_t \lambda + \nabla_3 \lambda$$

Así que, en conjunto, esto aparecería probablemente como una especie de corriente extra no asociada a los movimientos de las cargas eléctricas, o tal vez sería indistinguible de las corrientes eléctricas de alguna manera, salvo que impregna todo el espacio como una función continua. Parecería que hay corrientes en todas partes, en cierto sentido. Puedes ver por qué ni siquiera consideramos la existencia de tal campo. A menos que sea muy pequeño, lo habríamos detectado hace tiempo, ya que interactúa con el término fuente de la corriente eléctrica.

Un análisis del campo magnético pseudoescalar probablemente terminaría de la misma manera.


Entonces, ¿al tensor de Faraday le faltan en realidad dos componentes extra, un campo escalar y un campo pseudoescalar? Yo diría que no Pero si descubre lo contrario, probablemente ganará un premio Nobel. Buena suerte con eso. Como dije en la otra pregunta, no se deje engañar a pensar que, sólo porque $Fx$ tiene ocho componentes, faltan componentes del campo de Faraday. Es muy probable que no haya ningún componente que falte. Puedes ver esto considerando lo que harían esos componentes en las ecuaciones de Maxwell de vainilla, como he hecho aquí.


Edición: algunas correcciones sobre la relación entre este campo escalar y la fijación gauge.

Este campo escalar eliminaría la libertad de cambiar $A$ a través de transformaciones gauge, como $\lambda$ especificaría la divergencia de $A$ . Recordemos que la fijación del calibre se basa en la capacidad de realizar la transformación,

$$A \mapsto A + \nabla \chi$$

Para algún campo escalar $\chi$ . Esto puede hacerse porque $\nabla \wedge (A + \nabla \chi) = \nabla \wedge A = F$ , por lo que el campo EM no cambia.

Pero si $\nabla \cdot A = \lambda$ entonces añadiendo el gradiente de un campo escalar cambiar el valor de $\lambda$ de forma medible, salvo en los casos más sencillos:

$$\nabla \cdot (A + \nabla \chi) = \lambda + \nabla^2 \chi$$

Ahora, se limitaría a las funciones de calibre $\chi$ que son estrictamente armónicos. Los campos armónicos suelen ser los que surgen de alguna elección de condiciones de contorno, es decir, esto correspondería a alguna elección de condición de contorno, y la contribución de campo de las corrientes no cambiaría. Por supuesto, es un esfuerzo de imaginación considerar cómo se podría hacer esto razonablemente para la fijación de galgas. Y si se encontrara una transformación que preservara $\lambda$ no dejaría $F$ invariante en general.

Así que la supuesta existencia de esta función $\lambda$ tendría profundas consecuencias hacia la fijación del calibre. No lo prohíbe absolutamente, como yo pensaba en un principio, pero pone serias limitaciones a la fijación que probablemente ya habríamos encontrado.

0 votos

Gracias por proporcionar una excelente respuesta. Parece que este término escalar adicional que podría aparecer en el campo electromagnético rompe la conservación de la carga? ¿Es posible que este término escalar pueda ser descrito en términos del cuatro potencial?

0 votos

+1. Esta respuesta hace que quiera pasar más tiempo estudiando álgebra geométrica.

0 votos

@linuxfreebird: Sí, este campo escalar especificaría completamente $\nabla A$ y luego por el análogo de Minkowski del teorema de Helmholtz, que especificaría $A$ hasta las condiciones de contorno. La existencia de tal campo escalar se manifestaría en rompiendo la capacidad de hacer la fijación del calibre . Por ejemplo, normalmente se puede utilizar el calibre de Lorenz estableciendo $\nabla (\nabla \cdot A) = 0$ . Pero aquí, $\lambda = \nabla \cdot A$ y si $\lambda$ existiera, no tendrías a priori razón para poner su gradiente (y por tanto, sus términos de corriente asociados) a cero.

6voto

Esto es más bien un comentario extendido para abordar los comentarios a la respuesta de Kyle

Por ejemplo, si hubiera una componente temporal en el campo eléctrico y magnético campo

En un contexto relativista, los componentes del campo eléctrico y del magnético no son componentes de vector sino que son componentes de a campo tensorial de 2º rango; los campos eléctrico y magnético forman parte de un objeto geométrico, no dos.

De hecho, una pista de esto se encuentra en el hecho de que el campo magnético es, en 3D, un pseudovector en lugar de un campo vectorial.

Así que, de hecho, la pregunta "¿cuál es la componente temporal del campo eléctrico y magnético?" presupone en realidad una falsedad ; supone que los campos eléctricos y magnéticos son cuatro vectores separados pero relacionados.

Pero, no son .

Como un tensor de rango 2 tiene dos índices, podemos hablar propiamente del tiempo-tiempo componente, el tiempo-espacio componentes, y el espacio-espacio componentes del tensor pero no el tiempo o el componente(s) espacial(es).

0 votos

Por fin he podido responder a la siguiente pregunta: physics.stackexchange.com/questions/103664/ Por favor, investigue esta cuestión, porque hay un cuarto componente en el campo.

1voto

martinatime Puntos 1863

El Faraday $2$ -tensor $F$ que describe la intensidad del campo electromagnético es antisimétrica, por lo que no hay lugar en $1\!+\!3$ dimensiones para cualquier componente extra además del $3$ eléctrico y $3$ magnéticos. Esto es sencillo, pero por qué ¿es antisimétrico?

Se puede encontrar una pista sobre la naturaleza del campo electromagnético expresando las ecuaciones de Maxwell en términos del tensor de Faraday $F$ y el tensor de Maxwell $H$ : $$\begin{eqnarray*}\partial_{[i}F_{jk]} = 0\text{,}&\quad&\partial_{[i}H_{jk]} = J_{ijk} =_\text{def}\frac{1}{3!}\epsilon_{ijkl}J^l\text{.}\end{eqnarray*}$$ Físicamente, $F \Leftrightarrow (\mathbf{E},\mathbf{B})$ es la intensidad de campo y $H \Leftrightarrow (\mathbf{D},\mathbf{H})$ es la excitación electromagnética. La primera da la ley de Gauss para el magnetismo y la ley de inducción de Faraday, mientras que la segunda da las leyes de Gauss y de Ampère-Maxwell.

Lo más destacable de este formulario es que ninguno de las ecuaciones de Maxwell se preocupan por la métrica del espaciotiempo en absoluto. Más bien, la métrica aparece como parte de la estrella de Hodge en una ley separada que vincula los tensores de Maxwell y Faraday: $$H \propto \star F\text{,}$$ una buena manera de interpretar que es una relación constitutiva dando las propiedades dieléctricas y magnéticas del espaciotiempo. Por ejemplo, tanto la electrodinámica no lineal de Born-Infeld puede describirse mediante una relación constitutiva alternativa manteniendo las ecuaciones de Maxwell anteriores, como los efectos de las correcciones de vacío de primer orden de la QED derivadas por Heisenberg y Euler.

Si los tres campos eléctricos y magnéticos vectoriales provienen de la cuatrapotencial de cuatro componentes, ¿existe una cuarta componente del campo eléctrico y magnético?

Ahora vamos a flip la observación anterior en un argumento. El electromagnetismo no es gravedad, por lo que, si bien es posible que necesitemos la métrica en algún momento para convertirla en una teoría totalmente predictiva, deberíamos ser capaces de poner las ecuaciones que describen el campo electromagnético en sí en una forma independiente tanto de la métrica como de la conexión. Por tanto, las ecuaciones del electromagnetismo deberían seguir teniendo sentido aunque el espaciotiempo no tuviera ni métrica ni conexión. Lo que queda además de topología es _estructura diferencial_ .

Conclusión: El electromagnetismo debe ser describible mediante formas diferenciales, y las formas diferenciales corresponden a tensores antisimétricos covariantes. En $n$ dimensiones, el número de componentes independientes de un $k$ -forma es $C(n,k)$ . Por lo tanto, si sabemos que los campos eléctricos y magnéticos se entremezclan en las transformaciones a través de los marcos y por lo tanto deben ser partes del mismo tensor/ $k$ -forma, el número total de componentes para $n=4$ debe ser uno de $\{1,4,6\}$ .

Teniendo $4$ tanto para el campo eléctrico como para el magnético haría $8$ Así que eso es un no-go.


Por cierto, si ya sabemos también que la intensidad de campo $2$ -forma tiene un potencial, $F = \mathrm{d}A$ entonces $A$ debe ser un $1$ -forma y tiene cuatro componentes independientes... pero no deberíamos haber esperado que toda esa aparente libertad fuera física en primer lugar, porque $A\mapsto A+\chi$ para cualquier $1$ -forma $\chi$ con $\mathrm{d}\chi = 0$ produce lo mismo $F$ . Tenga en cuenta que $F = \mathrm{d}A$ implica que $\mathrm{d}F = 0$ que es la ley de Gauss para el magnetismo y la ley de inducción de Faraday.

0 votos

Por fin he podido responder a la siguiente pregunta: physics.stackexchange.com/q/103664 Por favor, investigue esta cuestión, porque hay un cuarto componente en el campo.

-1voto

numberwang Puntos 51

Las respuestas proporcionadas por Alfred Centauri y Kyle Kanos contienen afirmaciones fácticas, pero hay más geometría oculta en el problema. Es cierto que los vectores de campo eléctrico y magnético no son verdaderos vectores, sino más bien pseudovectores, como afirma Alfred Centauri, que pertenecen a tensores de campo de rango 2, como afirma Kyle Kanos. Sin embargo, Kyle Kanos mencionó que los campos provienen del cálculo exterior $\partial^\mu A^{\nu} - \partial^{\nu}A^{\mu}$ lo que sugiere que los campos EM pueden entenderse como posibles bivectores del espacio cuatridimensional https://en.wikipedia.org/wiki/Bivector . Utilizando el remapeo de un dual hodge https://en.wikipedia.org/wiki/Hodge_dual En este caso, se podrían convertir los componentes del bivector en cuatro componentes de un cuatro-vector del espacio-tiempo como forma de condensar la notación. Esto implicaría una posible cuarta componente del campo EM.

0 votos

El enlace del bivector incluye una discusión sobre el tensor de campo de E&M, mostrando que la componente temporal es el campo eléctrico, tal y como yo había afirmado.

5 votos

El dual de Hodge en 4 dimensiones sólo lleva bivectores a bivectores. En este caso, sólo da el tensor de campo EM dual.

0 votos

He publicado una pregunta adicional relacionada con ésta, relativa a los dos tensores electromagnéticos: physics.stackexchange.com/q/103601

i-Ciencias.com

I-Ciencias es una comunidad de estudiantes y amantes de la ciencia en la que puedes resolver tus problemas y dudas.
Puedes consultar las preguntas de otros usuarios, hacer tus propias preguntas o resolver las de los demás.

Powered by:

X